Những câu hỏi liên quan
Kiên Đỗ
Xem chi tiết
Vui Nguyen
Xem chi tiết
Ngọc Hạnh Nguyễn
Xem chi tiết
Nguyễn Tom
Xem chi tiết
tran duc huy
Xem chi tiết
Nguyễn Việt Lâm
28 tháng 11 2019 lúc 0:01

\(\frac{cosa}{1+sina}+\frac{sina}{cosa}=\frac{cos^2a+sina\left(1+sina\right)}{cosa\left(1+sina\right)}=\frac{1+sina}{cosa\left(1+sina\right)}=\frac{1}{cosa}\)

\(\frac{sin^2a+cos^2a+2sina.cosa}{\left(sina-cosa\right)\left(sina+cosa\right)}=\frac{\left(sina+cosa\right)^2}{\left(sina-cosa\right)\left(sina+cosa\right)}=\frac{sina+cosa}{sina-cosa}=\frac{\frac{sina}{cosa}+1}{\frac{sina}{cosa}-1}=\frac{tana+1}{tana-1}\)

\(\left(sin^2a\right)^3+\left(cos^2a\right)^3=\left(sin^2a+cos^2a\right)^3-3sin^2a.cos^2a\left(sin^2a+cos^2a\right)\)

\(=1-3sin^2a.cos^2a\)

\(sin^2a-tan^2a=tan^4a\left(\frac{sin^2a}{tan^4a}-\frac{1}{tan^2a}\right)=tan^4a\left(sin^2a.\frac{cos^2a}{sin^2a}-\frac{1}{tan^2a}\right)\)

\(=tan^4a\left(cos^2a-cot^2a\right)\) bạn ghi sai đề câu này

\(\frac{tan^3a}{sin^2a}-\frac{1}{sina.cosa}+\frac{cot^3a}{cos^2a}=tan^3a\left(1+cot^2a\right)-\frac{1}{sina.cosa}+cot^3a\left(1+tan^2a\right)\)

\(=tan^3a+tana-\frac{1}{sina.cosa}+cot^3a+cota\)

\(=tan^3a+cot^3a+\frac{sina}{cosa}+\frac{cosa}{sina}-\frac{1}{sina.cosa}\)

\(=tan^3a+cot^3a+\frac{sin^2a+cos^2a-1}{sina.cosa}=tan^3a+cot^3a\)

Khách vãng lai đã xóa
Trần Đức Thắng
Xem chi tiết
Trần Thị Loan
15 tháng 7 2015 lúc 22:19

A = (sin2a + cos2a)3 - 3sin2a. cos2a.(sin2a + cos2a) + 3sin2a.cos2a = 1 - 3sin2a. cos2a + 3sin2a. cos2a = 1

Nguyễn Hoàng Minh Quang
Xem chi tiết
My Kieu
Xem chi tiết
o0o I am a studious pers...
15 tháng 10 2016 lúc 19:51

\(Sin^6a+cos^6a+3\left(sin^2a+cos^2a\right)\)

\(=\left(sin^2a+cos^2a\right)^3\)

\(=1\)

\(\)

nam võ hoài
Xem chi tiết
Nguyễn Lê Phước Thịnh
25 tháng 2 2022 lúc 23:19

Đặt \(\sin^2\alpha=a;\cos^2\alpha=1\)

Theo đề, ta có: \(a^3+b^3=1-3ab\) và \(a+b=1\)

\(a^3+b^3+3ab=\left(a+b\right)^3-3ab\left(a+b\right)+3ab\)

\(=1^3-3ab+3ab=1\)

Do đó: \(a^3+b^3=1-3ab\)(đpcm)